[Math] basis of span with functions

linear algebravector-spaces

I'm not sure how to solve the following problem:

Determine a basis and the dimension of $\langle e^{2t}, t^2, t \rangle$ (in the space of functions from $\mathbb{R}$ to $\mathbb{R}$)

When finding the basis of a span of vectors, I create a matrix and reduce the matrix to row-echolon form. If there is a row without a pivot, I know that the corresponding vector is not part of the basis. Since this is a span of functions instead of vectors, I'm not sure how to solve this.

From what I have found on the exchange here, I know that, in order for this span to be a basis, it should satisfy the following:
$$a\cdot e^{2t}+b\cdot t^2+c\cdot t = 0 \quad\text{ if and only if }\quad a = b = c = 0$$

  1. Choosing $t = 0$, shows that $a$ has to be $0$
  2. Choosing $t = 1$, shows that $b = -c$ or $-b = c$

But step $2$ means that $b$ and $c$ can basically be any number right?
However, according to the answer sheet the given span should be a basis. What am I missing?

And is there a way I can solve questions like these with a matrix as well?

Best Answer

You can solve it with matrices, sort of.

To check linear independence of a set $\{f_1, f_2, f_3\}$ of twice differentiable functions on $\mathbb{R}$, you can calculate the Wronskian, the determinant defined as:$$W(f_1, f_2, f_3)(t) = \begin{vmatrix} f_1(t) & f_2(t) & f_3(t) \\ f_1'(t) & f_2'(t) & f_3'(t)\\ f_1''(t) & f_2''(t) & f_3''(t)\end{vmatrix}$$

Notice that if your functions satisfy $\alpha f_1 + \beta f_2 + \gamma f_3 = 0$ on $\mathbb{R}$, then linearity of the derivative also implies $$\alpha f_1' + \beta f_2' + \gamma f_3' = 0$$ $$\alpha f_1'' + \beta f_2'' + \gamma f_3'' = 0$$

so $W(f_1, f_2, f_3)(t)$ will be zero on $\mathbb{R}$, since the matrix columns will be dependent.

Thus, if $W(f_1, f_2, f_3)$ is not identically zero, we can conclude that $\{f_1, f_2, f_3\}$ are linearly independent. However, if $W(f_1, f_2, f_3) = 0$ you cannot outright conclude that $\{f_1, f_2, f_3\}$ is linearly dependent.

In your case, we have:

$$W(e^{2t}, t^2, t)(t) = \begin{vmatrix} e^{2t} & t^2 & t \\ 2e^{2t} & 2t & 1\\ 4e^{2t} & 2 & 0\end{vmatrix} = -2 e^{2 t} (2t^2 - 2 t + 1) \ne 0$$

for all $t \in \mathbb{R}$ so $\{e^{2t}, t^2, t\}$ is certainly linearly independent, and hence a basis for your space.